Difference between revisions of "2018 AMC 10A Problems/Problem 1"

m
m
Line 4: Line 4:
 
===Solution===
 
===Solution===
 
Evaluating the expressions starting with the innermost one, we get that the answer is <math>\boxed{\textbf{(B)}\ \frac{11}{7}}</math>
 
Evaluating the expressions starting with the innermost one, we get that the answer is <math>\boxed{\textbf{(B)}\ \frac{11}{7}}</math>
{MAA Notice}
+
{{MAA Notice}}

Revision as of 15:45, 8 February 2018

What is the value of \[\left(\left((2+1)^{-1}+1\right)^{-1}+1\right)^{-1}+1?\]$\textbf{(A) } \frac58 \qquad \textbf{(B) }\frac{11}7 \qquad \textbf{(C) } \frac85 \qquad \textbf{(D) } \frac{18}{11} \qquad \textbf{(E) } \frac{15}8$

Solution

Evaluating the expressions starting with the innermost one, we get that the answer is $\boxed{\textbf{(B)}\ \frac{11}{7}}$ The problems on this page are copyrighted by the Mathematical Association of America's American Mathematics Competitions. AMC logo.png